Real Analysis Definition and Explanation

Click For Summary
The discussion focuses on defining a subsequential limit in real analysis and explaining its implications. A subsequential limit, denoted as ##t## for the sequence ##(s_n)##, means there exists a subsequence that converges to ##t##. To rigorously define this, one must specify that there is a strictly increasing sequence of indices ##(n_k)## such that ##\lim s_{n_k}=t##. The conversation emphasizes the importance of precision in definitions, particularly regarding the relationship between subsequences and their limits. Overall, the discussion highlights the necessity of clarity in mathematical definitions and the logical structure of subsequential limits.
Mr Davis 97
Messages
1,461
Reaction score
44

Homework Statement


1) Suppose ##t## is a subsequential limit for ##(s_n)##. Write the precise definition of the meaning of this statement.

2) Explain why there exists a strictly increasing sequence ##(n_k)^\infty_{k=1}## of natural numbers such that ##\lim s_{n_k}=t##.

Homework Equations

The Attempt at a Solution


I am not exactly sure what to do for 1). Doesn't this just mean that there exist a subsequence that converges to t? I am not exactly sure how precisely I need to state it, or how I would if a lot of precision is necessary.

For 2), doesn't this just follow from the definition of a subsequential limit?
 
Physics news on Phys.org
Mr Davis 97 said:
For 2), doesn't this just follow from the definition of a subsequential limit?

Is it mentioned that the sequence ##\{s_n\}_{n=1}^\infty## is bounded?
 
Eclair_de_XII said:
Is it mentioned that the sequence ##\{s_n\}_{n=1}^\infty## is bounded?
Is is not mentioned that the sequence is bounded or unbounded.
 
If the sequence is ##\{s_n\}##, then a subsequence is written as ##\{s_{n_k}\}##, where ##n_1, n_2, n_3, \dots, ## is an increasing sequence of indices. Can you write a definition of a subsequence that converges to t using this notation?
 
Eclair_de_XII said:
Is it mentioned that the sequence ##\{s_n\}_{n=1}^\infty## is bounded?

This isn't relevant for the problem. The statement is true with or without boundedness.

Mr Davis 97 said:

The Attempt at a Solution


I am not exactly sure what to do for 1). Doesn't this just mean that there exist a subsequence that converges to t? I am not exactly sure how precisely I need to state it, or how I would if a lot of precision is necessary.

For 2), doesn't this just follow from the definition of a subsequential limit?

(1) Write it as formally as you can!
(2) We can't help you with (1) if you don't tell us what definition of subsequential limit you use.
 
Last edited by a moderator:
Mr Davis 97 said:

Homework Statement


1) Suppose ##t## is a subsequential limit for ##(s_n)##. Write the precise definition of the meaning of this statement.

Since you've recently asked questions about the logic of quantifiers, my guess is that this problem is testing your skill in writing definitions involving the use of "there exists".

2) Explain why there exists a strictly increasing sequence ##(n_k)^\infty_{k=1}## of natural numbers such that ##\lim s_{n_k}=t##.

As @Mark44 indicated, if you write a definition that says "there exists a subsequence", that statement is not identical to the claim that "there exists a strictly increasing sequence ##(n_k)## of natural numbers such that...". , although the latter statement might be taken for granted in a typical mathematical article. So imagine someone insists that you "split hairs" and explain why a definition that says "there exists a subsequence..." implies a statement about the existence of a certain strictly increasing sequence of natural numbers. (To do that, you need to use the formal definition of a subsequence.)
 
Question: A clock's minute hand has length 4 and its hour hand has length 3. What is the distance between the tips at the moment when it is increasing most rapidly?(Putnam Exam Question) Answer: Making assumption that both the hands moves at constant angular velocities, the answer is ## \sqrt{7} .## But don't you think this assumption is somewhat doubtful and wrong?

Similar threads

  • · Replies 4 ·
Replies
4
Views
2K
  • · Replies 9 ·
Replies
9
Views
2K
  • · Replies 1 ·
Replies
1
Views
2K
  • · Replies 7 ·
Replies
7
Views
2K
  • · Replies 12 ·
Replies
12
Views
2K
  • · Replies 6 ·
Replies
6
Views
2K
  • · Replies 2 ·
Replies
2
Views
2K
  • · Replies 6 ·
Replies
6
Views
2K
  • · Replies 5 ·
Replies
5
Views
1K
  • · Replies 11 ·
Replies
11
Views
3K